Đến nội dung

FanquanA1

FanquanA1

Đăng ký: 06-12-2011
Offline Đăng nhập: 10-12-2013 - 17:01
-----

#364537 $f(2x)=2f(x)$ và $f(f^2(x))=xf(x)$

Gửi bởi FanquanA1 trong 24-10-2012 - 20:26

Bài toán: Xác định các hàm số liên tục $f:\mathbb{R^+} \to \mathbb{R^+}$ thỏa mãn:

  • $f(2x)=2f(x),\forall x \in \mathbb{R^+}$
  • $f(f^2(x))=xf(x),\forall x \in \mathbb{R^+}$
  • $f(x) \in \mathbb{N^*},\forall x \in \mathbb{N^*}$.

Trước tiên từ 2 suy ra f đơn ánh và do f liên tục trên $\left ( 0,+\infty \right )$ nên f toàn ánh trên $\left ( 0,+\infty \right )$. Suy ra tồn tại $a\in (0.+\infty )$ sao cho $f(a)=1$. Thay a vào 2 ta có: $f(1)=a$. Thay $x=1$ vào 2 ta có: $f(a^2)=a$. Thay $x=a^2$ vào 2 ta có: $f(a^2)=a^3$. Suy ra $a=1$. Do f liên tục và đơn ánh nên f tăng hoặc giảm nhưng do 3 nên f tắng (Nếu f giảm thì cho $x\in N*$, x đủ lớn thì $f(x)$ không có giá trị xác định). Ta sẽ chứng minh$f(x)=x$, $x\in N^*$
Thay $x=1$ vào 1 ta có: $f(2)=2$. Thay $x=2$ vào 1 ta có: $f(4)=4$. Do $f(3)$ nguyên và f đơn ánh nên $f(3)=3$.
Giả sử khẳng định đúng đến n. Tức là $f(n)=n$. Ta sẽ chứng minh: $f(n+1)=n+1$. Thật vậy:
$f(2n)=2n$ và vì f đơn ánh nên: $n=f(n)<f(n+1)<...<f(2n-1)<f(2n)=2n$ suy ra: $f(n+1)=n+1$
Như vậy $f(n)=n,n\in N*$
Ta có: $f(m)=2f(\frac{m}{2})=2^nf(\frac{m}{2^n})\Rightarrow \frac{m}{2^n}=f(\frac{m}{2^n})$
Mặt khác: $\frac{m}{2^n}$ trù mật trên $\left ( 0,+\infty \right )$ nên:
$x_0=lim\frac{m}{2^n}=limf(\frac{m}{2^n})=f(lim\frac{m}{2^n})=x_0$
Vậy $f(x)=x,x\in R^+$


#354549 Tìm các số nguyên tố p; q sao cho $p^3 -q^5=(p+q)^2$

Gửi bởi FanquanA1 trong 16-09-2012 - 10:46

Tìm các số nguyên tố p; q sao cho $p^3 -q^5 = (p+q)^2$
Thank you very much!!!
----------------------------------------
Bạn tham khảo thêm về $\LaTeX$ tại:
http://diendantoanho...cong-thức-toan/
Tham khảo về cách đặt tiêu đề tại:
http://diendantoanho...i-khong-bị-xoa/
Mình m0ve bài này sang b0x số học THCS bạn nhé ;)

Nếu 1 trong 2 số không có số nảo bằng 3 thì:
TH1:$p\equiv q\equiv \pm 1(mod3)$ thì vế trái chia hết cho 3 còn vế phải không chia hết cho 3, vô lý.
TH2:$p\equiv -q\equiv \pm 1(mod3)$ thì vế phải chia hết cho 3 còn vế trái không chia hết cho 3. vô lý.
Như vậy 1 trong 2 số phải có 1 số bằng 3
Nếu $p=3$ thì vô lý vì $q^5+q^2+6q=18\Rightarrow q=2\vee 3$
Nếu $q=3$ thì $p^3-p^2-6p=252=7.36\Rightarrow p=7$


#351268 $\sqrt{x^2-p}+2{x^2-1}=x$

Gửi bởi FanquanA1 trong 31-08-2012 - 23:53

Giải phương trình
$$\sqrt{x^2-p}+2{x^2-1}=x$$
Với $p$ là hằng số thực.

ĐKXĐ: $x^2-p\geq 0$
Phương trình $\Leftrightarrow 4x^4-4x^3-4x^2+2x+1+p=0\Leftrightarrow x^4-x^3-x^2+\frac{x}{2}+\frac{1+p}{4}=0$ (1)
Đặt $x=y+\frac{1}{4}\Rightarrow (1)\Leftrightarrow y^4=ax^2+bx+c$ (2)
Trong đó: $a=\frac{11}{8},b=\frac{-1}{8},c=\frac{19-64p}{16}$
Gọi $d$ là nghiệm thực của phương trình bậc ba :$b^2=4(a+2d)(c+d^2)$
Xét: $f(x)=(a+2d)x^2+bx+c+d^2$ thì phương trình có nghiệm kép.
Nếu : $a+2d=0 \Rightarrow f(x)=c+d^2$
Nếu : $a+2d\neq 0 \Rightarrow f(x)=(a+2d)(x+\frac{b}{2(a+2d)})^2$
Như vậy: $(2)\Leftrightarrow x^4+2dx^2+d^2=f(x)\Leftrightarrow (x^2+d)^2=f(x)$
Nếu: $a+2d=0\Rightarrow x^2=-d\pm \sqrt{c+d^2}$
Nếu: $a+2d<0$ thì vô nghiệm
Nếu: $a+2d>0$ thì: $x^2\mp \sqrt{a+2d}(x-\frac{b}{2(a+2d)})+d=0$


#351250 Giải pt nghiệm nguyên dương

Gửi bởi FanquanA1 trong 31-08-2012 - 23:04

Giải pt nghiệm nguyên dương:$x^{2}y^{2}=z^{2}(z^{2}-x^{2}-y^{2})$

Ta thấy nếu $(x,y)=d (tương tự với các cặp khác) \Rightarrow z\vdots d$. Như vậy có thể giả sử $x,y,z$ đôi một nguyên tố cùng nhau.
$x^2y^2=z^2(z^2-x^2-y^2)\Leftrightarrow (x^2+z^2)(y^2+z^2)=2z^4\Rightarrow (x,z)>1\Rightarrow x=y=0$


#351242 Giải phương trình nghiệm nguyên không âm $(x;y)$ thỏa mãn: $...

Gửi bởi FanquanA1 trong 31-08-2012 - 22:41

Giải phương trình nghiệm nguyên không âm $(x;y)$ thỏa mãn:
$$33^x+31=2^y$$

Với $y=5,y=6$ thì $x=0,x=1$
Xét $y=7,8,9$ không thỏa mãn
Xét $y\geq 10$: $33^x+31=2^y\Rightarrow x+1\vdots 4$ (Sử dụng khaci triển nhị thức Newton)
Mặt khác: $32(33^{x-1}+...+33+1)=2^5(2^{y-5}-1)\Leftrightarrow 33^{x-1}+...+33+2=2^{y-5}\Rightarrow \frac{x(x-1)}{2}+2\vdots 4$
$\Rightarrow x-1\vdots 4$
Vô lý. Vậy chỉ có 2 nghiệm


#348529 China Girls Math Olympiad 2012

Gửi bởi FanquanA1 trong 20-08-2012 - 15:21

Hình đã gửi
Bài 3
Tìm mọi cặp số nguyên $(a,b)$ thỏa mãn tồn tại số nguyên $d>0$ sao cho $a^n+b^n+1$ chia hết cho $d$ với mọi số nguyên dương $n$.



Theo AoPS

Ta có: $a+b+1\vdots d,a^2+b^2+1\vdots d\Rightarrow (a+b)^2-(a+b)-2ab\vdots d\Rightarrow 2-2ab\vdots d$
Mặt khác:$(a+b+1)(a^2+b^2+1)\vdots d\Rightarrow a^3+b^3+ab(a+b)+a+b+a^2+b^2+1\vdots d$
$\Rightarrow 2+ab\vdots d\Rightarrow 4+2ab\vdots d$
Suy ra: $6\vdots d$
Nếu $d=1$ thì $(a,b)$ vô số
Nếu $d=2$ thì a,b chẵn lẻ khác nhau
Nếu $d=3$ thì $a\equiv b\equiv 1(mod3)$
Nếu $d=6$ thì $a\equiv b\equiv 1(mod3)$, a,b chẵn lẻ khác nhau


#348524 CMR: $\sum_{k=0}^n \dfrac{\binom{n}{k} ^2}{(2k+1)\bi...

Gửi bởi FanquanA1 trong 20-08-2012 - 14:57

Bài Toán :

Cho trước số nguyên dương $n$ ; chứng minh rằng :

$ \sum_{k=0}^{n} \dfrac{ \binom{n}{k} ^2}{(2k+1)\binom{2n}{2k}} = \dfrac{2^{4n} (n!)^4}{(2n)! (2n+1)!}$






Nguyễn Kim Anh


Áp dụng định lý nhị thức $(1+x)^u=\sum_{k=0}^{\infty }\binom{u}{k}x^k$ thì ta có: $\frac{1}{\sqrt{1-x^2}}=\sum_{n=0}^{\infty }\frac{\binom{2n}{n}}{4^n}x^{2n}$ ($\left | x \right |<1$)
Mặt khác: $arcsinx=\sum_{n=0}^{\infty }\frac{\binom{2n}{n}}{4^n}\frac{x^{2n+1}}{2n+1}$ (Sử dụng công thức Maclaurin, tuy nhiên việc chứng minh cái này hơi dài hoặc mọi người có thể xem tại http://vi.wikipedia..../Hàm_lượng_giác)
Xét hàm f xác định trong $(-1,1)$ $f(x)=\frac{arcsinx}{\sqrt{1-x^2}}$ và $\sum_{n=0}^{\infty }a_n.x^{2n+1}$ là khai triển của f
Ta có: $a_n=\sum_{k=0}^{n}\frac{\binom{2k}{k}}{(2k+1).4^k}\frac{\binom{2n-2k}{n-k}}{4^{n-k}}=\frac{(2n)!}{4^n(n!)^2}\sum_{k=0}^{n}\frac{\binom{n}{k}^2}{(2k+1)\binom{2n}{2k}}$
Do $(arcsinx)'=\frac{1}{\sqrt{1-x^2}}$ nên $(1-x^2)f'(x)=1+xf(x)$. Suy ra:
$1=\sum_{n=0}^{\infty }(2n+1)a_nx^{2n}-\sum_{n=1}^{\infty }(2n-1)a_{n-1}x^{2n}-\sum_{n=1}^{\infty }a_{n-1}x^{2n}=a_0+\sum_{n=1}^{\infty }((2n+1)a_n-2na_{n-1})x^{2n}$
Vì $a_0=1$ nên $\frac{a_n}{a_{n-1}}=\frac{2n}{2n+1},(\forall n\geq 1)$
Theo quy nạp ta sẽ có: $a_n=\frac{4^n(n!)^2}{(2n+1)!}$
Vậy ta có điều phải chứng minh


#348171 CMR: Trong 3 số a, b, c phải có 1 số bằng 2012.

Gửi bởi FanquanA1 trong 19-08-2012 - 10:53

Ta tổng quát bài toán bằng cách đặt $d=2012$. Giả sử $p=a+b+c-d$ là số nguyên tố
Ta có: $c\leq d\leq a$ và $d(ab+bc+ ca-d^2)-d^2(a+b+ c-d)\vdots p \Leftrightarrow d^3-(a+b+c)d^2+(ab+bc+ca)d-d^3=(d-a)(d-b)(d-b)\vdots p$
Suy ra p là ước của 1 trong các số $d-a$, $d-b$, $d-c$.
Nếu $a=c$ thì $a=b=c=d$
Nếu $a> c$ thì $a+b+2c>2d$ (AM-GM) $\Rightarrow p>d-c$, $p=a+b+c-d>a-d$
Suy ra p không là ước của $d-a$ và $d-c$ nên p chia hết $d-b$
Nếu $d>b$ thì do $p=a+b+c-d\geq 2d> d-b$ nên vô lý.
Nếu $b>d$ thì do $p> b-d$, vô lý.
Vậy $b=d$


#347093 Tìm tất cả các số nguyên $(m,n),m>n$ thỏa $[m^2+mn,mn-n^2]...

Gửi bởi FanquanA1 trong 16-08-2012 - 00:14

Tìm tất cả các số nguyên $(m,n),m>n$ thỏa $[m^2+mn,mn-n^2]+[m-n;mn]=2^{2005}$
Với $[a;b]$ là bội chung nhỏ nhất của $a;b$
Ivan Landjev

Đặt $d=(m,n)$ suy ra $m=d.a,n=d.b$,$(a,b)=1$ ta có: $\frac{mn(m^2-n^2)}{(m^2+mn,mn-n^2)}+\frac{mn(m-n)}{(mn,m-n)}=2^{2005}$ $\Leftrightarrow mn(m-n)(\frac{m+n}{(m^2+mn,mn-n^2)}+\frac{1}{(mn,m-n)})=2^{2005}$ . Ta có: $(m^2+mn,mn-n^2)=d^2(a(a+b),b(a-b))$
Nếu a,b cùng lẻ thì: $dmn(a-b)(\frac{d(a+b)}{2d^2}+\frac{1}{d})=2^{2005}\Leftrightarrow d^2ab(a-b)(a+b+2)=2^{2006}$
Do a,b lẻ nên $a=b=1$ vô lý
Nếu a,b khác tính chắn lẻ thì:$dmn(a-b)(\frac{d(a+b)}{d^2}+\frac{1}{d})=2^{2005}\Leftrightarrow d^2ab(a-b)(a+b+1)=2^{2005}$
Do $a-b$ lẻ nên $a=b+1$. Thay vào ta có: $d^2.b(b+1)^2=2^{2004}\Leftrightarrow b=1$. Suy ra: $d=2^{1000},a=2^{1001},b=2^{1000}$


#346707 Tìm 1<k<10 $x^2+ky^2=z^2$, $kx^2+y^2=t^2$ có nghi...

Gửi bởi FanquanA1 trong 14-08-2012 - 17:45

Giải như sau:
Bổ đề: $p \in \mathbb{P}, p \equiv 3 \pmod{4}, a^2+b^2 \vdots p \Leftrightarrow p|a,b$
Ta có $(k+1)(x^2+y^2)=z^2+t^2$
Vì $1<k<10 \Rightarrow 2<k+1<11 \Rightarrow k+1=3,4,5,6,7,8,9,10$
Thấy nếu $k+1=3,6,7$ áp dụng bổ đề dễ dàng suy ra vô lý bằng cách lùi vô hạn
Do đó $k+1=4,5,8,9,10 \Rightarrow k=3,4,7,8,9$
Nếu $k=3,8$ suy ra $x^2+ky^2=z^2$ và $kx^2+y^2=t^2$ rõ ràng có nghiệm $(x,y)=(1,1)$
Nếu $k=7$ suy ra $x^2+7y^2=z^2$ và $7x^2+y^2=t^2$ có nghiệm $(x,y)=(3,1)$
Giờ ta chỉ xét $k=4,9$ nhưng hai Th này quả thực khó khăn, mong mọi người trợ giúp, nó đã đưa về phương trình Pytago nhưng đến đây mọi chuyện vẫn chưa xong vì ta có tới hai phương trình
Quả thực là hơi khó!!

Bài này khi tìm được k thì ta sử dụng đẳng thức : $(x^2+y^2)(z^2+t^2)=(xz+yt)^2+(xt-yz)^2$
Như vậy với $k = 4 , 9$ thì $k+1=1^2+2^2 , k+1=1^2+3^2$


#334171 Giải phương trình nghiệm nguyên $7x-y^3=4k$

Gửi bởi FanquanA1 trong 10-07-2012 - 21:44

Nguyenta98 suy nghĩ bài sau xem chứng minh được không cùng anh nhé?

Cho (a,b)=1;Giả sử phân tích $a+b=p_1^{a1}p_2^{a2}...p_m^{a_m}$ liệu có luôn tồn tại một $p_i,i=1,2,...,m$ sao cho $a^n+b^n$ , n>2, không chia hết cho $p_i^2$ nếu bài này trả lời là có thì bài toán Fecma đã giải xong

Sẽ tồn tại nếu $ n=2^k $. Mình có đề nghị về Fecma nữa là bài toán tìm nghiệm nguyên dương: $a^p=b^p+c^p+kabc$ , $(a,b)=(b,c)=(c,a)=1$ , $ b+c>a>b>c $ ,$ p\in P$


#323117 Giải phương trình nghiệm nguyên dương:${x^{{y^x}}} = {y^{{x^y}}}$

Gửi bởi FanquanA1 trong 07-06-2012 - 14:11

Đề bài:Giải phương trình nghiệm nguyên dương
\[{x^{{y^x}}} = {y^{{x^y}}}\]
Ngoài ra nếu bạn nào có hứng thú có thể chứng minh bài toán sau đây
Với mọi số thực $x,y$ mà $y>x>0$ thì
\[{x^{{y^x}}} < {y^{{x^y}}}\]

Bài này có thể giải bằng cách khác ngắn hơn
Ta thấy : $f(x)=\frac{lnx}{x}$ nghịch biến nên $\frac{lnx}{x}>\frac{lny}{y}$ với $y>x>e$
suy ra: $x^y>y^x$ mà $x<y$ nên ta có đpcm
với $x=2$ nếu $y>3$ thì tương tự như trên. Nếu $y=3$ thì chỉ cần thay vào thử thấy đúng liền


#290802 Cho $p \ge 5$ nguyên tố sao cho $a^{2} + ab +b^{2} \...

Gửi bởi FanquanA1 trong 29-12-2011 - 17:36

Giả thiết $a^2+ab+b^2\vdots p$ là không cần thiết. Dùng khai triển nhị thức Newton ta có
$(a+b)^p=a^p+\textrm{C}_{p}^{1}a^{p-1}b+...+\textrm{C}_{p}^{p-1}ab^{p-1}+b^p$
$\textrm{C}_{p}^{p-k}=\dfrac{p!}{(p-k)!k!}$
Do $k\geq 1$ nên $(p-k)!k!$ không chia hết cho p nên $\textrm{C}_{p}^{p-k}=\dfrac{p!}{(p-k)!k!}\vdots p$
Suy ra :$(a+b)^p-a^p-b^p\vdots p$